« first day (2147 days earlier)      last day (2787 days later) » 

10:03 PM
@BernardMeurer Ok, you have to be more careful.
 
@0celo7 yeah?
 
@BernardMeurer The thing you said is trivial, isn't. $2^a$ is not even if $a<0$.
And $3^b$ is not odd if $b<0$.
 
Because odd and even are for integers, and those are not integers.
 
user218912
@0celo7 for infinitesimal rotations you take the limit as the rotation angle goes to $0$? or what?
 
10:07 PM
How can I rigorously say that though?
 
Furthermore, you should give a proof that $3^b$ is odd (read: not even) for $b=1,2,3,\dotsc$.
@Danu I know. I wanted @BernardMeurer to tell me that.
 
Don't worry, I don't get what he said
 
@Danu Do you agree that if one is to give a very pedantic proof, there is more work?
 
$$\frac{\log 3}{\log 2} \notin \mathbb{Q} \\ \text{Let's say} \frac{a}{b}=\frac{\log 3}{\log 2}; a,b\in\mathbb{Z}; (a,b)=1 \\ 2^{\frac{a}{b}}=3 \\ 2^a = 3^b \\ \text{It is clear that }2^a\text{ will always be even, while }3^b\text{ will always be odd}$$
 
Not really, no---all the issues you raise are one-liners.
@BernardMeurer So everything works up until the last sentence
 
10:10 PM
@Danu A pedantic proof includes the one-liners.
That's what I mean by pedantic here.
 
@0celo7 What? That 2 doesn't divide $3^b$ is really obvious, what more of a "proof" could one possibly give for that?
 
You should consider some separate cases for $a,b$
@0celo7 Sure, but they're not more work.
 
@ACuriousMind Uniqueness of prime factorization.
 
Lolwut
Just write $(2k+1)(2m+1)$
Much easier
Any product of odd things is odd
 
Dude I don't need to prove the commutativeness of addition every time I use it, I don't need to prove the uniqueness of prime factorization
 
user218912
10:12 PM
does $\eta_{\mu\mu}$ make any sense?
 
You don't need prime factorization!
 
Unless my whole point is to prove the uniqueness of prime factorization of course
 
@BernardMeurer Uh, uniqueness of prime factorization is nontrivial.
 
@IceLord Depends---in Einstein convention, NO!
 
user218912
@Danu alright then i need a dummy index.
 
10:13 PM
Maybe you don't need prime factorization.
 
100% you don't. I just gave you a proof.
 
@Danu He's making a very pedantic point that you don't know that $3\cdot 3$ is not divided by $2$ without appealing to the uniqueness of prime factorization.
 
But product of odds is odd? How does one prove that?
 
@0celo7 Write out $(2k+1)(2m+1)$
Notice that $1$ is odd
 
@Danu Is every odd of the form $2n+1$?
Seems like you need long division for that.
 
10:14 PM
@0celo7 If you're not willing to accept that then I am not interested in your proof ideas
 
user218912
@0celo7 yes
 
@0celo7 wtf
 
@ACuriousMind Am I wrong? Does one not need either (a) long division or (b) prime factorization?
 
user218912
@Danu does raising/lowering with a mixed index metric still the same as matrix multiplication?
 
user218912
10:16 PM
or what is the matrix form of the mixed metric?
 
@IceLord I don't like to think about tensors in terms of matrix multiplication much.
 
user218912
@Danu i need to for this case.
 
IIRC, only things with one raised and one lowered index really work like matrices
 
@0celo7 You're not technically wrong, but you're taking a different (and unreasonably low) amount of prior knowledge as granted here.
 
user218912
then why does the regular metric work like a matrix?
 
10:17 PM
You probably get away with all of that stuff in Minkowski space anyways
 
@ACuriousMind Good. That's all I wanted to hear.
 
All I know is that there are some tricky annoying things---like the fact that the determinant of a matrix is basis-independent, but the determinant of the metric is not (in GR).
 
@IceLord What seems to be the issue?
 
@0celo7 I would urge you to rethink your approach here though, since I don't think you said anything that actually helped @BernardMeurer.
 
@ACuriousMind I'm looking at this like I was taught to in my algebra class.
I still don't think one should say it's trivially true that $2^a\ne3^b$.
 
10:19 PM
I would urge you to either say something constructive or just stop wasting Bernard's time
 
I'm willing to accept that $3^b$ is odd.
 
@0celo7 How would you prove that $3^b$ is odd?
 
@BernardMeurer Sorry, don't want to waste your time.
 
user218912
@0celo7 nothing I figured it out, just have one question is $\eta_{\mu\beta} \alpha^\mu_{\, \,\nu} = \alpha_{\beta\nu}$ or $\alpha_{\nu\beta}$ ?
 
@0celo7 Which is different from the intro to analysis context that BM is in. In algebra one emphasizes things that are "obvious" in the reals because they fail in more general algebraic structures, while in the intro context this type of exercise is used to learn proof structures like proof by contradiction or induction while using arithmetic the students are already familiar with.
 
10:21 PM
@BernardMeurer I told you twice, already
Prove that a product of two odd numbers is always odd
@IceLord Depends on where the higher $\mu$ is in the initial thing.
 
user218912
@Danu what do you mean? I fixed it.
 
As you have it now, you should make it $\alpha_{\beta\nu}$
 
user218912
alright thanks
 
In general, though, you usually define tensors with their indices in a certain way
And then you just define the raised or lowered versions by an explicit equation involving the metric
So you just choose what you want to do
 
user218912
@Danu can you answer my question by before as well? I asked is an infinitesimal rotation the same as taking the angle of rotation as the limit goes to $0$?
 
10:24 PM
Typically the best thing is to make the convention so that you preserve horizontal order of indices
@IceLord Sure, small angle
 
@IceLord cc @Danu That's ambiguous phrasing once again. As you're saying it, it sounds as if you want to take $\lim_{\theta\to 0} R(\theta)$ where $R(\theta)$ is the matrix that rotates by $\theta$, which is wrong
 
user218912
@ACuriousMind yes that's what I want to do
 
user218912
why is it wrong?
 
Try it.
You'll see.
 
user218912
I did and I get the wrong answer.
 
10:26 PM
What did you get?
 
user218912
how is it supposed to be?
 
user218912
I get the reverse of what it is supposed to be.
 
What, exactly, did you get for $\lim_{\theta\to 0} R(\theta)$?
 
user218912
1 0
 
user218912
0 1
 
10:27 PM
:P
 
user218912
do I not know my trig functions properly
 
Yes, the identity, that's correct. How is that the "opposite of what it is supposed to be"?
 
user218912
or am i doing it wrong
 
user218912
@ACuriousMind it's supposed to be
 
user218912
0 -1
 
user218912
10:28 PM
1 0
 
That looks a bit similar, but isn't the "opposite" in any meaningful sense.
 
user218912
sorry bad wording
 
user218912
anyway what is going on?
 
user218912
mind I ask
 
First take the general rotation matrix, and Taylor expand in the angle
 
10:29 PM
@IceLord Well, how did you define "infinitesimal rotation"?
 
user218912
limit $\theta \to 0$
 
Limit of what?
 
user218912
$R(\theta)$
 
user218912
you said that's wrong
 
Yeah, that is not right
 
10:30 PM
@Danu I'd prefer not giving an answer until we've got a precise question, because not asking the right question has been a recurring problem here
 
user218912
xP
 
@IceLord Yes, it's wrong. Where did you get that definition from?
 
@ACuriousMind I don't know IceLord at all
 
user218912
@ACuriousMind my intuition.
 
Aha.
 
user218912
10:30 PM
@Danu I'm Icosahedron/3075.
 
Okay.
 
user218912
the precise question is
 
user218912
how do you do infinitesimal rotations given a matrix $R(\theta)$
 
Well, but what's your definition of "infinitesimal rotation"?
 
user218912
a baby rotation
 
user218912
10:32 PM
like a really small one
 
The one you gave is non-sensical, because it'll always be the identity. And you know what it's "supposed" to be, but how can you know what something is supposed to be when you haven't even defined it?
 
user218912
@ACuriousMind the right matrix is in cahill.
 
user218912
he doesn't derive it from the rotation matrix though.
 
user218912
just shows it
 
user218912
apparently it's obvious
 
user218912
10:34 PM
but i never learned this
 
See, now we're getting closer. So you're reading a book. There are three possibilities: 1. The book is really bad. 2. The book somewhere defines this but you skipped over it. 3. The book is way over your head and you should be reading something simpler.
 
user218912
@ACuriousMind it's always 2
 
Sooooo... what do you learn from that? :P
 
@IceLord So go back and read the book more closely!
I'm not going to tell you the answer because that would reinforce you relying on asking people here instead of reading carefully.
 
user218912
oh
 
user218912
10:44 PM
you taylor expand and add it to the identity so the second term is the derivative causing the trig functions to switch giving the right answer.
 
user218912
right @ACuriousMind ?
 
I don't know what "causing the trig functions to switch" means nor do I know why you "add it to the identity". How did you come up with that?
 
user218912
I found a formula for infinitesimal transformations
 
user218912
I mean the identity is part of the expansion
 
@ACuriousMind Ok, point taken.
@ACuriousMind lol my thoughts exactly
 
user218912
10:53 PM
there's something wrong with me 0.o
 
Yes, but that's unrelated to this.
 
user218912
what do you mean?
 
I'm probably trolling.
 
user218912
troll.
 
@ACuriousMind sorry
I'm hoarding the glory
@BernardMeurer You should say "suppose $\frac{\log 2}{\log 3}=\frac{a}{b}$" instead of "let's say"
 
11:02 PM
Already changed
 
Ok. Also that tower of stuff isn't really nice. If you're typing out homework, consider writing full sentences.
 
user218912
@ACuriousMind I searched everywhere in the book, it doesn't teach how to do it.
 
@IceLord What do you want...
 
user218912
how do you do infinitesimal rotations?
 
user218912
or transformations for that matter
 
user218912
11:09 PM
what's the formula?
 
@IceLord What do you want...
No need to bug @ACuriousMind
no need to bug @ACuriousMind
whoa lag
 
user218912
did you see my message?
 
Nice, two "no need to bug ACM" pings :D
 
two?
My internet is freaking out.
 
user218912
refresh the page
 
user218912
11:10 PM
you'll see
 
@ACuriousMind sorry, didn't want to bug you
@ACuriousMind sorry
lol
what's happening
@ACuriousMind do you have ping noises on
@IceLord Which one?
 
Yes.
 
user218912
@0celo7 how do you do infinitesimal transformations?
 
user218912
what is the formula
 
@ACuriousMind How do your ears exist? The noise is so loud!
How do you do them?
@IceLord New rule: if it's in Shankar and I can find it in 5 minutes, I will not help
 
user218912
11:13 PM
@0celo7 oh i found it
 
@ACuriousMind did I used to be like this?
@IceLord Read chapter 11 in Shankar.
@IceLord What about Cahill though?
 
@0celo7 It's not that loud
 
@ACuriousMind Well you survived Wacken so that's understandable.
@ACuriousMind When we write a rotation operator in terms of Pauli matrices, what is that actually called?
 
I'm not sure what you mean
 
Is it a group representation?
 
11:18 PM
Or rather, if I am, then I don't think there's a word for that
@0celo7 The act of choosing the Pauli matrices to be rotations is choosing a representation, yes
Writing an arbitrary rotation in those Pauli matrices is then, well, "writing the representation of that rotation".
 
Right. Just checking.
 
Oh, I though you were searching for a catchier term :P
 
@ACuriousMind No, just trying to make sense out of physics.
I want to learn about representation theory, but I feel like I'll learn the wrong things.
I asked my advisor for a rep theory book and he thought I wanted to classify the homogeneous spaces...
(I do, that's why I'm reading Helgason. But I want to learn physics rep theory too.)
 
I think the standard recommendation for more physically relevant rep theory is Fulton and Harris?
 
@ACuriousMind He said "that's too algebraic" when I asked him about it.
His copy is relegated to the unreachable, use-once-a-decade shelf.
 
11:24 PM
@0celo7 Is that from a physics or from a geometer's standpoint?
 
@ACuriousMind From a geometer, of course.
But is that what the mathematical physicists use?
@ACuriousMind So, when we do the SHO stuff in string theory, are we assuming no degeneracy of the string number operator?
@ACuriousMind According to Shankar, one can show that there is no degeneracy in 1-dim bound states.
 

« first day (2147 days earlier)      last day (2787 days later) »